分布を制限しています


8

ましょうバツ IIDの配列であるN01ランダム変数。定義S0=0及びSn=k=1nXkためのn11の極限分布を見つける

1nk=1n|Sk1|(Xk21)

この問題は、中心極限定理の章にある、確率論の問題集にあります。

以降Sk1Xk、独立しているE(|Sk1|(Xk21))=0

V|Sk1|バツk21=ESk12バツk212=ESk12Eバツk212=2k1

ことに注意してください|Sk1|バツk21は明らかに独立していない。問題は、確率におけるシリヤエフの問題からです、それ自体が同じ著者の教科書に基づいているにあります。教科書は相関変数のCLTをカバーしていないようです。どこかに静止したミキシングシーケンスが隠れているかどうかわかりません...

シミュレーションを実行して答えを感じました

import numpy as np
import scipy as sc
import scipy.stats as stats
import matplotlib.pyplot as plt

n = 20000 #summation index
m = 2000 #number of samples

X = np.random.normal(size=(m,n))
sums = np.cumsum(X, axis=1)
sums = np.delete(sums, -1, 1)
prods = np.delete(X**2-1, 0, 1)*np.abs(sums)
samples = 1/n*np.sum(prods, axis=1)

plt.hist(samples, bins=100, density=True)
x = np.linspace(-6, 6, 100)
plt.plot(x, stats.norm.pdf(x, 0, 1/np.sqrt(2*np.pi)))
plt.show()

以下は2000年サンプルのヒストグラムです(=20.000)。それはかなり正規分布に見えます...

ここに画像の説明を入力してください


@MartijnWeterings私はしばらくの間問題を熟考していて、行き詰まっているので、私はこれを投稿しました。それはおそらく些細なことから遠いです...
ガブリエルロモン

@MartijnWeterings 、したがってV | S k 1 |X 2 k1 = E S 2 k 1X 2 k1 2E|Sk1|バツk21=0V|Sk1|バツk21=ESk12バツk212
Gabriel Romon

@MartijnWeteringsはい、自明な平等を省略しましたのためのx R ...|バツ|2=バツ2バツR
ガブリエルRomon

シミュレーションのヒストグラムは、正規分布のひどい一致です。確信が持てない場合は、尖度を計算してください。
whuber

@MartijnWeteringsはい、コードで恥ずかしい省略をしました。ヒストグラムだけでなく、通常のヒストグラムに更新しました。分散の正確な値がわかりますか?
ガブリエルロモン

回答:


1

分布をシミュレートすると、ラプラス分布に似たものが得られます。さらに良いのは、q-Gausian(理論を使用して見つける必要がある正確なパラメーター)のようです。

あなたの本には、それに関連するCLTのバリエーションが含まれている必要があると思います(qの一般化された中心極限定理、おそらくセクション7.6にありますが従属変数の和の中心極限定理ですが、私はそれを調べることができません。利用可能な本を持っていない)。

シミュレーション

library(qGaussian)
set.seed(1)
Qstore <- c(0) # vector to store result

n <- 10^6  # columns X_i
m <- 10^2  # rows repetitions

pb <- txtProgressBar(title = "progress bar", min = 0,
                     max = 100, style=3)
for (i in 1:100) {  
  # doing this several times because this matrix method takes a lot of memory
  # with smaller numbers n*m it can be done at once

  X <- matrix(rnorm(n*m,0,1),m)
  S <- t(sapply(1:m, FUN = function(x) cumsum(X[x,])))
  S <- cbind(rep(0,m),S[,-n])
  R <- abs(S)*(X^2-1)
  Q <- t(sapply(1:m, FUN = function(x) cumsum(R[x,])))

  Qstore <- c(Qstore,t(Q[,n]))
  setTxtProgressBar(pb, i)
}
close(pb)

# compute histogram 
x <- seq(floor(min(Qstore/n)), ceiling(max(Qstore/n)), 0.2)
h <- hist(Qstore/(n),breaks = x)

# plot simulation
plot( h$mid, h$density, log = "y", xlim=c(-7,7),
      ylab = "log density" , xlab = expression(over(1,n)*sum(abs(S[k-1])*(X[k]^2-1),k==1,n) ) )

# distributions for comparison
lines(x, dnorm(x,0,1),                   col=1, lty=3)      #normal 
lines(x, dexp(abs(x),sqrt(2))/2,         col=1, lty=2)      #laplace
lines(x, qGaussian::dqgauss(x,sqrt(2),0,1/sqrt(2)), col=1, lty=1)      #qgauss

# further plotting
title("10^4 repetitions with n=10^6")
legend(-7,0.6,c("Gaussian", "Laplace", "Q-Gaussian"),col=1, lty=c(3,2,1),cex=0.8)

教科書の内容については、第1 第2 をご覧ください。問題は、第3.4章
Gabriel Romonで

@GabrielRomonはこれらのリンクをありがとうございました。それを見て、私の電話から、正規分布ではないq-Gaussianまたは他の制限分布について何も見つけることができませんでした。だから、どちらかの分布は>> ^ 6私たちは前に10非常に遅い収束nを有する参照して、または質問が章に適合しない(ブックからそれをですが、私はどちらも質問を見つけることができませんでした?)。高次モーメントのプロット(nの関数として)は、変換がまだ発生するかどうかをよりよく示すかもしれませんが、これは典型的なCLTのケースではないと思います。
Sextus Empiricus

1
これは問題集の問題3.4.14 です。
ガブリエルロモン
弊社のサイトを使用することにより、あなたは弊社のクッキーポリシーおよびプライバシーポリシーを読み、理解したものとみなされます。
Licensed under cc by-sa 3.0 with attribution required.